Consider the following transactions: Sold goods on credit for 2000 Received a payment from a credit customer of 1300 Sold goods for cash for 4000 Sold goods on credit for 1500 After these transactions, the balance of the receivables account is: Select one: O a. 3500 in debit c b. 2200 in credit O c. 2200 in debit O d. 3500 in credit

Answers

Answer 1

The balance of the receivables account after the transactions is 3500 in debit.

The balance of the receivables account after these transactions can be calculated as follows:

Received from a credit customer reduces the accounts receivable amount by $1300.

So, we now have a balance of $700 in accounts receivable account.

Sold goods on credit for $2000 and $1500.

So the total amount of goods sold on credit = $2000 + $1500

                                                                          = $3500.

This amount is added to the accounts receivable account balance.

Now the balance of accounts receivable account = $700 + $3500

                                                                                  = $4200.

The balance of the receivables account is now $4200, which is debit in nature.

Therefore, the correct answer is 3500 in debit.

Learn more about balance of the receivables account from the given link:

https://brainly.com/question/24871345

#SPJ11


Related Questions

Market Research conducted a survey to find out whether people who earn more money purchase more expensive goods. The following graph indicates the relationship between income the survey subjects earned and the price of the home that they purchased. 500 450 400 350 300 PRICE (Thousands of dollars per home) Slope: 2.5 Y-Intercept: 50 200 150 100 50 0 90 100 10 20 30 00 50 50 70 00 INCOME (Thousands of dollars per year) 300 250 PRICE (Thousands of dollars Slope: 2.5 Y-Intercept: 50 200 150 100 0 0 10 100 20 40 70 INCOME (Thousands of dollars per year) The variable shown on the horizontal axis i price The units for the variable on the vertical axis are thousands of dollars per year There are two ways to view the information presented on the graph. First, the graph tells us the amount a person with a certain income is likely to spend on a home, and second, it tells us the probable income of a person who spent a certain amount on a home. For example, if an individual earned $60,000 last year and purchased a new home, you would expect that person to have paid about_$250 for the home. Similarly, if someone just paid $200,000 for a home, you could use this graph to estimate that this person's income was probably around $60,000

Answers

The given graph illustrates the relationship between income and the price of homes purchased. It provides two perspectives: estimating the amount a person with a certain income is likely to spend on a home and estimating the probable income of a person who spent a certain amount on a home.

For example, if an individual earned $60,000 last year and purchased a new home, based on the graph's trendline, it would be expected that the person paid around $250,000 for the home. Similarly, if someone recently paid $200,000 for a home, the graph suggests that their income was likely around $60,000.

The slope of the graph, which is 2.5, indicates that for every increase of $1,000 in income, the price of the home increases by $2,500. The y-intercept of 50 represents the baseline price for homes when income is zero.

By analyzing the graph, we can make estimations and observations about the relationship between income and home prices.

Learn more about interpreting graphs and relationships between income and purchasing patterns here:

brainly.com/question/31795933

#SPJ11

Blanchard Company manufactures a single product that sells for $160 per unit and whose total variable costs are $120 per unit. The company’s annual fixed costs are $629,000. The sales manager predicts that annual sales of the company’s product will soon reach 39,900 units and its price will increase to $199 per unit. According to the production manager, variable costs are expected to increase to $139 per unit, but fixed costs will remain at $629,000. The income tax rate is 25%. What amounts of pretax and after-tax income can the company expect to earn from these predicted changes? Prepare a forecasted contribution margin income statement.

Answers

Answer:

Following are the solution to the given question:

Explanation:

                                                  Income statement

sales                                                   [tex]39900\times 199 \ \ \ \ \ \ \ \ \ \ \ \ \ \ \ \ \ \ \ \ \ \ \ \ \ \ \ \ 7940100\\\\[/tex]

The less average cost of variable [tex]39900\times 139 \ \ \ \ \ \ \ \ \ \ \ \ \ \ \ \ \ \ \ \ \ \ \ \ \ \ \ \ 5546100 \\\\[/tex]

margin for contribution                                                                       [tex]2394000\\\\[/tex]

Lesser fixed costs                                                                                  [tex]629000\\\\[/tex]

Income from of the company or tax                                                   [tex]1765000 \\\\[/tex]

Lower-income tax by [tex]25\%[/tex]                                                                      [tex]441250 \\\\[/tex]  

after-tax revenue                                                                                    [tex]1323750\\\\[/tex]

A rightward shift in a demand curve and a leftward shift in a supply curve both result in a
Select one:
a. Higher equilibrium price.
b. Lower equilibrium quantity.
c. Lower equilibrium price.

Answers

A rightward shift in a demand curve and a leftward shift in a supply curve both result in a Higher equilibrium price. The correct option is a.

A shift to the right in a demand curve indicates that there is more demand for the product being measured. This change shows that customers are more likely to buy more of the product at each price point. A leftward shift in a supply curve, on the other hand denotes a reduction in a product's supply. This change implies that producers are supplying less of the product at each price point.

The equilibrium price, which is the price at which supply and demand converge, will rise in both situations. The increased equilibrium price reflects either a rise in demand or a fall in supply for the good. The correct option is a.

Learn more about demand curve at:

brainly.com/question/13131242

#SPJ4

If the price level falls, then: (a) The Aggregate Expenditure (AE) curve will shift up and there will be a movement along the Aggregate Demand curve in the direction of higher output. (b) The Aggregate Expenditure (AE) curve will shift up and the Aggregate Demand (AD) curve will shift to the right. (c) The Aggregate Expenditure (AE) curve will shift down and the Aggregate Demand (AD) curve will shift to the left. (d) The Aggregate Expenditure (AE) curve will shift down and there will be a movement along the Aggregate Demand curve in the direction of lower output.

Answers

If the price level falls, then the Aggregate Expenditure (AE) curve will shift up and there will be a movement along the Aggregate Demand curve in the direction of higher output. Option A is the correct answer.

The primary factor influencing supply and demand in the economy is the cost of products and services. Yet, the opposite is also true: variations in demand and supply have an effect on the cost of products and services. Option A is the correct answer.

The relationship between overall price levels and total demand is not always obvious or straightforward. However, in the broadest sense, rising aggregate demand is correlated with rising price levels. The entire amount of completed products and services that are wanted in an economy is measured by aggregate demand. The total amount of money traded for such products and services at a certain price point and period is how this measurement is stated.

Learn more about Demand here:

https://brainly.com/question/1245771

#SPJ4

Which of the following best describes the meaning of the multiplicities next to the number 4 in the preceding diagram?
1-Each stock is bought/sold at least once.
2-Each stock is bought/sold only once.
3-Each stock is bought/sold a maximum of many times.
4-Each stock is bought/sold a minimum of many times.
5-Both "Each stock is bought/sold at least once" and "Each stock is bought/sold a maximum of many times"

Answers

The correct answer is 5 - "Both 'Each stock is bought/sold at least once' and 'Each stock is bought/sold a maximum of many times'".

The multiplicities next to the number 4 indicate the number of times each stock is bought or sold.

The first part of the statement, "Each stock is bought/sold at least once," implies that every stock in the diagram has been involved in at least one transaction, meaning it has been bought or sold. The second part, "Each stock is bought/sold a maximum of many times," suggests that the maximum number of transactions for any stock is indicated by the multiplicity next to the number 4.

Therefore, the correct answer is 5, as it combines both aspects: each stock is involved in at least one transaction and can be involved in multiple transactions, up to the maximum indicated by the multiplicity of 4.

To know more about stock:

https://brainly.com/question/31940696

#SPJ4

Consider again Mockingbird Airlines’ problem as described in the overbooking case study in Chapter 9.of Making Hard Decisions.
Construct a simulation model of the system, and use it to find Mockingbird’s optimal policy regarding overbooking. Compare this answer with the one based on the analysis done in Chapter 9.

Answers

The optimal policy regarding overbooking for Mockingbird Airlines in the simulation model may differ from the one based on the analysis in Chapter 9.

The simulation model provides a more accurate representation of the system dynamics and allows for experimentation with different policies, considering various factors such as passenger no-show rates, cancellation rates, and revenue. By running the simulation multiple times with different policies, it is possible to find the policy that maximizes Mockingbird Airlines' expected profit. This approach takes into account the inherent uncertainties and variability of the system, providing a more robust and realistic solution compared to the analysis in Chapter 9, which relies on assumptions and simplifications.

Learn more about profit here:

https://brainly.com/question/31117493

#SPJ11

The central bank targets the real policy rate, i.e., r = Tt, by setting the nominal interest rate it accordingly, with rt = it - π and π = Tt-1. The approximate Phillips curve is given - with expectations formation as stated - by: πt = πt-1 + (m+z) - ax ut, with the specific values of the markup m = 0.025, the catch-all variable z = 0.015 and a = 0.5 in this exercise. The labor force is given by L = 5,000. For the time being, the government follows a balanced budget policy, i. e., G = Tt. The risk premium equals t = 0.03 and the central bank sets the nominal interest rate at Tt = 0.01. (b) Calculate the natural rate of unemployment un, natural output Y, and transform - using the specified values - the Phillips curve from the format given above into the format given in Chapter 9 of the slides and the book, i.e., πt Tt-1 = × (Yt - Yn).

Answers

The natural rate of unemployment the natural rate of unemployment, or simply the natural rate, is temperament defined as the unemployment rate that prevails when the labor market is in equilibrium. The inflation rate does not affect the natural unemployment rate.

The equation that governs the Phillips curve is given byπt = πt-1 + (m+z) - ax utwhere πt is the current rate of inflation, πt-1 is the lagged rate of inflation, u is the unemployment rate, m is the markup, z is the catch-all variable, and a is the parameter that governs the slope of the Phillips curve.

The Phillips curve can be written in terms of expected inflation, rather than the lagged rate of inflation.πt - E(πt+1)

= (m+z) - au twhere E(πt+1) is the expected inflation rate over the next period.The natural rate of unemployment can be calculated by setting πt

= E(πt+1), which yields un

= (m+z)/a

= (0.025+0.015)/0.5

= 0.08.The natural rate of output can be calculated as Yn = L(1-un)

= 5,000(1-0.08)

= 4,600.(b) Transformation of the Phillips CurveThe Phillips curve in the format given above can be written in the form πt Tt-1

= (m+z)/(aTt) (Yt - Yn) -where the natural rate of unemployment.

To know more about Temperament visit:

https://brainly.com/question/31365144

#SPJ11

The present value of $2000 to be received 5 years from now, assuming a required rate of return of 5% a. 1567 O b. 2552.56 O c. 2100 O d. 2000

Answers

The present value is approximately $1567. Option A is the correct answer.

The present value of $2000 to be received 5 years from now, assuming a required rate of return of 5%, can be calculated using the formula for present value. The formula is:

Present Value = Future Value / (1 + Rate)^n

Where:

Future Value = $2000

Rate = 5% or 0.05

n = 5 years

Plugging in the values into the formula:

Present Value = $2000 / (1 + 0.05)^5

Calculating this expression, we find that the present value is approximately $1567. Option A is the correct answer.

You can learn more about present value at

https://brainly.com/question/30390056

#SPJ11

Timberly Construction makes a lump-sum purchase of several assets on January 1 at a total cash price of $800,000. The estimated market values of the purchased assets are building, $460,800: land, $307,200; land improvements, $28.800; and four vehicles. $163,200. Required: 1-a. Allocate the lump-sum purchase price to the separate assets purchased. 1-b. Prepare the journal entry to record the purchase. 2. Compute the first-year depreciation expense on the building using the straight-line method, assuming a 15-year life and a $30.000 salvage value. 3. Compute the first-year depreciation expense on the land improvements assuming a five-year life and double-declining-balance depreciation.

Answers

1-a. To allocate the lump-sum purchase price to the separate assets purchased, we need to determine the proportionate value of each asset relative to the total estimated market value of all the assets.

Total estimated market value of the assets = $460,800 + $307,200 + $28,800 + $163,200 = $960,000

Allocation of the purchase price:

Building:

Proportionate value = ($460,800 / $960,000) x $800,000 = $384,000

Land:

Proportionate value = ($307,200 / $960,000) x $800,000 = $256,000

Land improvements:

Proportionate value = ($28,800 / $960,000) x $800,000 = $24,000

Vehicles:

Proportionate value = ($163,200 / $960,000) x $800,000 = $136,000

1-b. The journal entry to record the purchase would be:

Date: January 1

Assets:

Building                                  $384,000

Land                                        $256,000

Land Improvements              $24,000

Vehicles                                $136,000

Cash                                         $800,000

2. To compute the first-year depreciation expense on the building using the straight-line method, we need to subtract the salvage value from the cost and divide it by the useful life.

Cost of the building = $384,000Salvage value = $30,000Useful life = 15 yearsDepreciation expense per year = (Cost - Salvage value) / Useful lifeDepreciation expense per year = ($384,000 - $30,000) / 15 = $21,600

Therefore, the first-year depreciation expense on the building using the straight-line method would be $21,600.

3. To compute the first-year depreciation expense on the land improvements using the double-declining-balance method, we need to determine the depreciation rate and apply it to the initial cost.

Cost of the land improvements = $24,000Useful life = 5 yearsDepreciation rate = 2 / Useful life = 2 / 5 = 40%First-year depreciation expense = Depreciation rate x Initial costFirst-year depreciation expense = 40% x $24,000 = $9,600

Therefore, the first-year depreciation expense on the land improvements using the double-declining-balance method would be $9,600.

About Market

Markets are the institutions, procedures, social relations and infrastructure where businesses sell goods, services and labor for people in exchange for money. Goods and services sold using legal means of payment such as fiat money. This activity is a part of economic.

Learn More About Market at https://brainly.com/question/28267513

#SPJ11

Today is t = 0. You have just bought a five-year zero-coupon Treasury bond with $1,000 face value. You paid $920.
(a) What is the annually compounded yield to maturity on the bond?
(b) Suppose that yields at all maturities increase to 2% immediately after you have purchased the bond. Calculate the annualized holding period return if you sell the bond one year after you have purchased it, at t = 1.
(c) Suppose instead that you did not sell the bond at t = 1 and held it to maturity. What is the annualized holding period return for the five year investment?

Answers

a. The annually compounded yield to maturity on the bond is 1.7%.

b. The annualized holding period return after one year is 8.7%.

c.  If the bond is held to maturity, the annualized holding period return for the five-year investment would be the same as the yield to maturity calculated in part (a), which is 1.7%.

(a) To calculate the annually compounded yield to maturity (YTM) on the bond, we can use the formula:

YTM = ((Face Value / Purchase Price) ^ (1 / Years to Maturity)) - 1

In this case, the face value of the bond is $1,000, and the purchase price is $920. The bond has a maturity of five years. Plugging in these values into the formula:

YTM = (($1,000 / $920) ^ (1 / 5)) - 1

= (1.08695652174 ^ 0.2) - 1

= 0.017 - 1

= 0.017 or 1.7%

Therefore, the annually compounded yield to maturity on the bond is 1.7%.

(b) If yields at all maturities increase to 2% immediately after purchasing the bond and you sell it after one year, the annualized holding period return can be calculated using the formula:

Holding Period Return = ((Selling Price / Purchase Price) ^ (1 / Holding Period)) - 1

Since the bond is a zero-coupon bond, the selling price after one year would be the face value of $1,000. Plugging in the values:

Holding Period Return = (($1,000 / $920) ^ (1 / 1)) - 1

= (1.08695652174 ^ 1) - 1

= 0.08695652174 or 8.7%

Therefore, the annualized holding period return after one year is 8.7%.

(c) If the bond is held to maturity, the annualized holding period return for the five-year investment would be the same as the yield to maturity calculated in part (a), which is 1.7%.

Holding the bond to maturity means receiving the face value of $1,000 at the end of the five-year period, and the return is based on the initial investment of $920. The return is earned through the discount or difference between the purchase price and the face value.

For more such questions on maturity

https://brainly.com/question/31393741

#SPJ8

Which one of the following actions is NOT an attractive option for trying to lower production costs per pair produced at one of your company's plants?
a. Reducing the number of branded models/styles produced from 350 to 250.
b. Increased spending for enhanced styling and features for branded footwear.
c. Reducing the use of superior materials.
d. Installing plant upgrade.
e. Increasing piecework incentive pay.

Answers

If you are looking to reduce the cost of production per pair of shoes made in your factory, spending more on improving the design and functionality of branded shoes based on the following measures is not an attractive option.

Option b is correct .

Adding features or investing in design improvements usually comes at additional costs in materials, labor, and design. While this increases the value and attractiveness of footwear, it is not a means of reducing costs.

:

Using cheaper materials can reduce costs, but the impact on quality and customer perception must be carefully considered. Plant modernization leads to increased efficiency, increased productivity and reduced costs in the long run.  

Hence, Option b is correct .

To know more about production costs visit :

https://brainly.com/question/15235684

#SPJ4

If a firm has at least some control over the price of its product, then the fimm cannot be in which market model: a. Oligopoly b. Pure monopoly c. Pure competition d. Monopolistic competition

Answers

Each company in a market with perfect competition is a price taker, which means that it has no control over the price. So, C is the best choice.

A monopolist is seen as a price maker since it has the power to determine the cost of the goods it sells. Demand, often known as consumer willingness and ability to buy the commodity, places restrictions on the monopolist.

Because they are price takers, businesses in a completely competitive market have no control over the cost of their goods. In a market with perfect competition, supply and demand forces control pricing. In a perfectly competitive market, businesses can only modify their output in an effort to increase profits.

To learn more about monopolist , click here.

https://brainly.com/question/31640700

#SPJ4

3. Answer the following questions about your own experience in the labor force a. When you or one of your friends is looking for a part-time job, how many weeks does it typically take? After you find a job, how many weeks does it typically last? 3 weeks spent looking/interviewing 26 weeks typically seasonal or during summers. b. From your estimates, calculate (in a rate per week) your rate of job finding f, and your rate of job separation, s. (Hint: the average spell of unemployment = 1/0) c. What is the natural rate of unemployment for the population you represent?

Answers

a, The rate of job finding is approximately 0.33 jobs per week, and b, the rate of job separation is approximately 0.04 jobs per week. The natural rate of unemployment is around 0.04 jobs per week.

a. Typically, it takes around 3 weeks to find a part-time job when either I or my friends are looking  in the labor force. Once a job is found, it typically lasts for about 26 weeks, especially if it is seasonal or during the summer.

b. To calculate the rate of job finding (f) and the rate of job separation (s), we can use the formulas

Rate of job finding (f) = 1 / time taken to find a job = 1 / 3 weeks = 1/3 per week

Rate of job separation (s) = 1 / average duration of a job = 1 / 26 weeks = 1/26 per week

c. The natural rate of unemployment for the population represented would be the difference between the rate of job separation and the rate of job finding. Therefore, the natural rate of unemployment can be calculated as

Natural rate of unemployment = s - f = (1/26) - (1/3) per week = 0.04 per week

To know more about labor force:

https://brainly.com/question/13962639

#SPJ4

ct. Bank reconciliation Prepare a bank reconciliation as of October 31 from the follow- Exercise 5.7 ing information: LO 3 ✓a. The October 31 cash balance in the general ledger is $1,688. ✓✓b. The October 31 balance shown on the bank statement is $746. ✓c. Checks issued but not returned with the bank statement were No. 462 for $26 and No. 483 for $100. -d. A deposit made late on October 31 for $900 is included in the general ledger bal- ance but not in the bank statement balance. Returned with the bank statement was a notice that a customer's check for $150 that was deposited on October 25 had been returned because the customer's account was overdrawn. f. During a review of the checks that were returned with the bank statement, it was noted that the amount of Check No. 471 was $64 but that in the company's records supporting the general ledger balance, the check had been erroneously recorded as a payment of an account payable in the amount of $46. X4 46 18

Answers

The reconciled balances per books and per bank statement should now be equal.

Bank Reconciliation: Bank Reconciliation is a comparison between the cash balance in the company's accounting records and the balance on the bank statement. The primary objective of preparing a bank reconciliation is to identify the differences between the two balances and make corrections where necessary. Hence, it helps in ensuring the accuracy of the company's financial records.  Steps to prepare bank reconciliation as of October 31 are as follows: Add any deposits in transit to the balance per books Deduct any outstanding checks from the balance per books Adjust the balance per books for any bank errors. Compare the adjusted balance per books to the balance per bank statement Add or deduct any bank memoranda to the balance per bank statement. The reconciled balances per books and per bank statement should now be equal.

To know more about Bank Reconciliation visit;

https://brainly.com/question/15525383

#SPJ11

At
what price does Monica sell her handbags to the independent retail
channnel distributor?
11. Should Monica propose the Grand* Mart deal as suggested? Or should she take a pass and stay exclusively with the independent retailer channel? Or should she renegotiate the initial 2,000 bag deal

Answers

The information provided does not include specific details about Monica's handbags, their production cost, or the terms of the proposed deal with Grand Mart. Without this information, it is not possible to determine the price at which Monica should sell her handbags to the independent retail channel distributor.

Regarding whether Monica should propose the Grand Mart deal, take a pass and stay exclusively with the independent retailer channel, or renegotiate the initial 2,000 bag deal, it depends on various factors such as the potential benefits and risks associated with each option, Monica's business goals and priorities, and the specific terms and conditions of the proposed deals. A thorough evaluation of these factors and careful consideration of Monica's business strategy and objectives is necessary to make an informed decision.

To know more about benefits visit-

brainly.com/question/31575296

#SPJ11

Make an accounting equation for the following question.

a. Amod tarted the business with $40,000
b. Ahmed purchased office equipement with a $3000
c. Almed purchased supplies on account from Accounts payable (Lux foods) on account for $500

Answers

The accounting equation represents the relationship between a company's assets, liabilities, and owner's equity. The equation is Assets = Liabilities + Owner's Equity. Let's make an accounting equation for the following question:

a. Amod started the business with $40,000The transaction indicates that Amod has invested $40,000 as capital to start the business. Therefore, the accounting equation would be Assets = Liabilities + Owner's EquityCash = 0 + $40,000b. Ahmed purchased office equipment for $3000.The transaction indicates that the office equipment is a company asset. Therefore, the accounting equation would be:Assets = Liabilities + Owner's EquityCash + Office Equipment = 0 + $40,000 + $3,000Cash + Office Equipment = $43,000c.

Almed purchased supplies on account from Accounts Payable (Lux Foods) on account for $500.The transaction indicates that the company has an accounts payable balance of $500. Therefore, the accounting equation would be: Assets = Liabilities + Owner's EquityCash + Office Equipment = $0 + $43,000Accounts Payable = $500The final accounting equation for these transactions would be: Assets = Liabilities + Owner's EquityCash + Office Equipment = $0 + $43,000Accounts Payable = $500.

Almed acquired supplies from Lux Foods and incurred a liability of $500, which will be recorded in their accounts payable. The accounts payable account represents the amount owed to creditors or suppliers for goods or services received on credit. Almed will need to make the payment to Lux Foods at a later agreed-upon date, typically within an established credit term.

to know more about business here;

brainly.com/question/15826679

#SPJ11

Why should you check the spelling by reading a document even if you used Writer's Spell Check feature? A. The Spell Check dictionary is usually out of date. B. Spell Check won't recognize mistakes you've made when you misspell a word in a way that makes another word. C. Spell Check only checks the first half of a document, so you have to check the rest yourself. D. The Spell Check suggestions are always wrong.

Answers

Answer:

B) Spell Check won't recognize mistakes you've made when you misspell a word in a way that makes another word.

Explanation:

X Corp. is located in State A and also has income from its business in State B. Residence State A tax rate is 22%. Source State B tax rate is 27%. X Corp. has income of 20,000 USD in State A, and 13.000 USD in State B.
Required: Taking into account above information and using the ordinary tax credit method, calculate the corporate income tax paid in residence State A by X Corp. (10 points)

Answers

X Corp. would have paid $890 in corporate income tax in residence State A using the ordinary tax credit method, taking into account the income earned in both State A and State B and the respective tax rates in each state.

To calculate the corporate income tax paid by X Corp. in residence State A using the ordinary tax credit method, we need to determine the taxable income in each state and apply the respective tax rates. Here's the breakdown:

Income in State A: $20,000

Income in State B: $13,000

Calculate the tax liability in State A:

Taxable income in State A: $20,000

State A tax rate: 22%

Tax liability in State A: $20,000 * 0.22 = $4,400

Calculate the tax liability in State B:

Taxable income in State B: $13,000

State B tax rate: 27%

Tax liability in State B: $13,000 * 0.27 = $3,510

Determine the amount of tax credit for taxes paid to State B:

Tax credit for State B taxes: Lesser of State B tax liability or income in State B

Tax credit for State B taxes: Min($3,510, $13,000) = $3,510

Calculate the corporate income tax paid in residence State A:

Tax paid in State A: Tax liability in State A - Tax credit for State B taxes

Tax paid in State A: $4,400 - $3,510 = $890

for more such questions on income

https://brainly.com/question/28390284

#SPJ11

Acklac Corporation uses the weighted-average method in its process costing system. This month, the beginning inventory in the first processing department consisted of 700 units. The costs and percentage completion of these units in beginning inventory were: Cost Percent Complete Materials costs.......... Conversion costs. $12,000 65% $2,000 10% A total of 9,200 units were started and 8,300 units were transferred to the second processing department during the month. The following costs were incurred in the first processing department during the month: Materials costs $200,800 $225,500 Conversion costs. The ending inventory was 75% complete with respect to materials and 15% complete with respect to conversion costs. A. The total cost transferred from the first processing department to the next processing department during the month is closest to: B. The cost of ending work in process inventory in the first processing department according to the company's cost system is closest to:

Answers

The total cost transferred from the first processing department to the next processing department during the month is closest to $335,650, and the cost of ending work in process inventory in the first processing department according to the company's cost system is closest to $20,078 for materials and $32,472 for conversion.

In the given question, Acklac Corporation uses the weighted-average method in its process costing system. This month, the beginning inventory in the first processing department consisted of 700 units. The costs and percentage completion of these units in beginning inventory were:

CostPercent CompleteMaterials costs = $12,000 65%

Conversion costs = $2,000 10%

A total of 9,200 units were started and 8,300 units were transferred to the second processing department during the month. The following costs were incurred in the first processing department during the month:

Materials costs = $200,800

Conversion costs = $225,500

The ending inventory was 75% complete with respect to materials and 15% complete with respect to conversion costs.

Calculation of total cost transferred from the first processing department to the next processing department during the month is shown below:

Calculation of equivalent units for material and conversion:

CostsMaterialsConversionBeginning Inventory $12,000 $2,000To account for700 × 65%

700 × 10%Work done during the period200,800 225,500To account for8,500 × 100% 8,500 × 100%

Ending inventory20,600 33,750To account for(700 × 75%) + (9,200 - 8,300) × 100%(700 × 15%) + (9,200 - 8,300) × 100%

Equivalent units29,040 27,570

Calculation of total cost for material and conversion:

CostsMaterialsConversionBeginning Inventory$12,000 $2,000

Cost added during the period200,800 225,500

Cost of goods to be accounted for212,800 227,500

Cost per equivalent unit for the period7.3181 8.2472

Total cost for units completed and transferred213,722 227,028

Cost of ending work in process inventory20,078 32,472

Total cost transferred from the first processing department to the next processing department during the month

= Cost of goods transferred to next department

= $213,722 + $227,028 - $20,078 - $32,472

= $388,200 - $52,550

= $335,650

Thus, the total cost transferred from the first processing department to the next processing department during the month is closest to $335,650.

Calculation of cost of ending work in process inventory in the first processing department according to the company's cost system:

CostsMaterialsConversionBeginning Inventory$12,000 $2,000

Cost added during the period200,800 225,500

Cost of goods to be accounted for212,800 227,500

Cost per equivalent unit for the period7.3181 8.2472

Total cost for units completed and transferred213,722 227,028

Cost of ending work in process inventory20,078 32,472

Therefore, the total cost transferred from the first processing department to the next processing department during the month is closest to $335,650, and the cost of ending work in process inventory in the first processing department according to the company's cost system is closest to $20,078 for materials and $32,472 for conversion.

Learn more about the process inventory from the given link-

https://brainly.com/question/30618969

#SPJ11




The non-bank private sector holds $250 of currency and $8,000 in deposits. Banks hold $1,000 in currency as reserves. In this economy, the money multiplier is 6.3 6.6 None of the above. 07

Answers

None of the above.  Without the reserve requirement ratio, it is not possible to determine the money multiplier in this economy based on the given information.

To calculate the money multiplier, we need to use the formula:

Money Multiplier = 1 / Reserve Ratio

To determine the money multiplier, we need to divide the total money supply by the monetary base. In this case, the monetary base consists of the currency held by the non-bank private sector and the reserves held by banks.

The currency held by the non-bank private sector is given as $250, and the reserves held by banks are $1,000. Therefore, the monetary base is $250 + $1,000 = $1,250.

However, we are not given the total money supply in this economy. Without knowing the total money supply, we cannot calculate the money multiplier.

The reserve ratio is the portion of deposits that banks are required to hold as reserves. In this case, the reserve ratio is not provided, so we cannot determine the money multiplier.

However, we can calculate the total reserves held by banks by adding the currency held as reserves ($1,000) and the currency held by the non-bank private sector ($250). So, the total reserves would be $1,000 + $250 = $1,250.

We can also calculate the total deposits in the economy by adding the currency held by the non-bank private sector ($250) and the deposits held by the non-bank private sector ($8,000). So, the total deposits would be $250 + $8,000 = $8,250.

The money multiplier can then be calculated as the ratio of total deposits to total reserves:

Money Multiplier = Total Deposits / Total Reserves

Money Multiplier = $8,250 / $1,250

Money Multiplier = 6.6

The correct money multiplier for the given scenario is 8, which differs from the options provided.

To know more about economy , visit;

https://brainly.com/question/28210218

#SPJ11

12 01:26:39 Fill in the blank: The manager of a(n). Mutiple Choice cost center profit center Investment center has control over costs and revenues in their department, but not over the use of i stment

Answers

Fill in the blank: The manager of an Investment center has control over costs and revenues in their department, but not over the use of investment. An investment center is a unit or division within an organization that is responsible for generating both revenues and costs.

The manager of an investment center has control over costs and revenues in their department but not over the use of investments. The manager of an investment center is accountable for the profits, capital, and investment utilization of the center. A profit center is responsible for generating revenue but is not responsible for allocating expenses or making investments.

A cost center is responsible for managing expenses and keeping them as low as possible. The manager of a cost center is accountable for the cost of running the department. The manager of a cost center, on the other hand, does not generate income for the organization. An investment center is responsible for managing revenue, expenses, and investments.

The manager of an investment center is accountable for the profit, capital, and investment utilization of the center. Investment centers have a high level of autonomy and are typically managed by senior executives. Because investment centers manage their own investments, they must evaluate the cost of capital and take risks.

As a result, investment centers have a greater potential for generating profits and contributing to the success of the organization.

To know more about generating visit :

https://brainly.com/question/12841996

#SPJ11

Which of the following best characterizes the source of European immigrants during the age of mass migration?
a. By the turn of the 20th century, Eastern and Southern Europeans had replaced Northern Europeans as the dominant immigrant groups.
b. Throughout the period Northern Europeans were the dominant immigrant group.
c. Throughout the period Southern Europeans were the dominant immigrant group.
d. By the turn of the 20th century, Northern Europeans had replaced Eastern and Southern Europeans as the dominant immigrant group.

Answers

"By the turn of the 20th century, Eastern and Southern Europeans had replaced Northern Europeans as the dominant immigrant groups." best characterizes the source of European immigrants during the age of mass migration. The correct option is B.

There was a significant change in the origin of European immigrants during the age of mass migration, which mostly took place between the middle of the 19th and the beginning of the 20th century. Initially Northern European nations like Germany, Ireland and the United Kingdom were home to the largest immigrant groups. However there was a noticeable change in the make up of immigrant groups as time went on.

By the turn of the 20th century, immigrants from Eastern and Southern Europe had begun to overtake those from Northern Europe. Several factors including the economic opportunities in the United States, the political unrest and social factors in their home countries, were responsible for this shift. During this time, nations like Italy, Poland, Russia and Greece emerged as major hubs for European immigration. The correct option is B.

Learn more about mass migration at:

brainly.com/question/4670470

#SPJ4

The IVplast Corporation uses an injection molding ma- chine to make a plastic product, 239, after receiving firm orders from its customers. IVplast estimates that it will receive 40 orders for 239 during the coming year. Each order of Z39 will take 120 hours of machine time. The annual machine capacity is 6,800 hours. 1. Calculate (a) the average amount of time that an order for 239 will wait in line before it is processed and (b) the average manufacturing cycle time per order for Z39. 2. IVplast is considering introducing a new product, Y28. The company expects it will receive 20 orders of Y28 in the coming year. Each order of Y28 will take 50 hours of machine time. Assuming the demand for 239 will not be affected by the introduction of Y28, calculate (a) the average waiting time for an order received and (b) the average manufacturing cycle time per order for each product, if IVplast introduces Y28.

Answers

a) The average waiting time for Y28 = (2.75 ÷ 0.364) − 50 = 31.58 hours and b) The average cycle time for Y28 = processing time + waiting time = 50 hours per order + 31.58 hours per order = 81.58 hours per order.

(a) The average amount of time that an order for 239 will wait in line before it is processed

Waiting time is calculated using Little’s law, which states that the expected number of units in the queue equals the expected time in queue multiplied by the expected processing rate.

The expected number of units in the queue = traffic intensity × (average cycle time + average waiting time)

For 239, the expected number of units in the queue = λ × W = (40 orders per year) ÷ (365 days per year) × (120 hours per order) = 4.11

The expected processing rate = C ÷ T = 6800 hours per year ÷ (40 orders per year × 120 hours per order) = 1.79

The traffic intensity (ρ) = λ × T = (40 orders per year) ÷ (365 days per year) × (120 hours per order) ÷ (6800 hours per year) = 0.18

The average waiting time (W) = ρ × (average cycle time) ÷ (1 − ρ) = 0.18 × [(120 hours per order) + (0.5 × 120 hours per order)] ÷ (1 − 0.18) = 33.5 hours

Therefore, the average amount of time that an order for 239 will wait in line before it is processed = 33.5 hours

(b) The average manufacturing cycle time per order for 239

The average cycle time for 239 = processing time + waiting time = 120 hours per order + 33.5 hours per order = 153.5 hours per order.

2. IV plast is considering introducing a new product, Y28.

The company expects it will receive 20 orders of Y28 in the coming year. Each order of Y28 will take 50 hours of machine time. Assuming the demand for 239 will not be affected by the introduction of Y28, calculate

(a) the average waiting time for an order received and (b) the average manufacturing cycle time per order for each product, if IVplast introduces Y28.

(a) The average waiting time for an order received

Let’s first calculate the expected number of units in the queue for each product.

For Z39, the expected number of units in the queue = traffic intensity × (average cycle time + average waiting time) = 0.18 × (120 + 33.5) = 24.93

For Y28, the expected number of units in the queue = traffic intensity × (average cycle time + average waiting time) = 0.055 × (50 + 0) = 2.75

The expected processing rate for Z39 is 1.79, and for Y28 it is 0.364Therefore, the average waiting time for Z39 = (24.93 ÷ 1.79) − 120 = 0.52 hours

The average waiting time for Y28 = (2.75 ÷ 0.364) − 50 = 31.58 hours

(b) The average manufacturing cycle time per order for each product, if IVplast introduces Y28.

The average cycle time for Z39 = processing time + waiting time = 120 hours per order + 0.52 hours per order = 120.52 hours per order

The average cycle time for Y28 = processing time + waiting time = 50 hours per order + 31.58 hours per order = 81.58 hours per order.

To know more about manufacturing  visit:

https://brainly.com/question/32717570

#SPJ11

Which of the following is an example of a critical incident question? O Tell me about a time when you put in extra effort to finish a project on time" O "If the CEO was here right now and asked you how to improve the company, what would you say? O Would you be willing to travel to South Africa two or three times a year? O "How would you handle an employee who is habitually 5 or 10 minutes late for his shift? QUESTION 3 Surveys of interviewers and small business owners showed that when these groups were presented with a list of llegal job int mistakenly o hardly any of them O about half of them a large majority

Answers

The example of a critical incident question from the options provided is: "How would you handle an employee who is habitually 5 or 10 minutes late for his shift?"

A critical incident question is designed to assess a candidate's ability to handle specific workplace situations or challenges. It aims to elicit responses that reveal the candidate's problem-solving skills, decision-making abilities, and their approach to handling difficult scenarios. In this case, the question about handling an employee who is habitually late for their shift is an example of a critical incident question because it presents a specific workplace situation that requires the candidate to provide a thoughtful and strategic response. The question assesses the candidate's potential conflict resolution skills, communication abilities, and their understanding of managing employee behavior in a professional setting.

Critical incident questions are valuable in the interview process as they allow employers to gauge a candidate's practical experience and problem-solving capabilities. By presenting real or hypothetical scenarios, employers can assess how candidates would respond to challenging situations in the workplace. These questions provide insight into a candidate's decision-making processes, interpersonal skills, and their ability to handle conflict or difficult employee situations effectively.

To know more about Employee,visit:
https://brainly.com/question/16952744
#SPJ11

Beckman Engineering and Associates (BEA) is considering a change in its capital structure. BEA currently has $20 million in debt carrying a rate of 7%, and its stock price is $40 per share with 2 million shares outstanding. BEA is a zero growth firm and pays out all of its earnings as dividends. The firm's EBIT is $13.327 million, and it faces a 40% federal-plus-state tax rate. The market risk premium is 5%, and the risk-free rate is 4%. BEA is considering increasing its debt level to a capital structure with 45% debt, based on market values, and repurchasing shares with the extra money that it borrows. BEA will have to retire the old debt in order to issue new debt, and the rate on the new debt will be 8%. BEA has a beta of 0.9.

What is BEA's unlevered beta? Use market value D/S (which is the same as wd/ws) when unlevering. Round your answer to two decimal places.

Answers

To determine BEA's unlevered beta, we must first understand the concept of levered beta and unlevered beta. The term "levered beta" refers to the risk associated with a company that has debt in its capital structure, while "unlevered beta" refers to the risk associated with a company that does not have debt in its capital structure.

Levered beta can be calculated using the following formula:

βL = βU[1 + (1 - T) * D/E]

Where:
βL is the levered beta
βU is the unlevered beta
T is the tax rate
D is the amount of debt in the company's capital structure
E is the amount of equity in the company's capital structure

To find BEA's unlevered beta, we can rearrange the above formula as follows:

βU = βL / [1 + (1 - T) * D/E]

Given the following information:

- BEA currently has $20 million in debt carrying a rate of 7%.
- BEA has a beta of 0.9.
- BEA is considering increasing its debt level to a capital structure with 45% debt, based on market values, and repurchasing shares with the extra money that it borrows.
- BEA will have to retire the old debt in order to issue new debt, and the rate on the new debt will be 8%.
- BEA is a zero-growth firm and pays out all of its earnings as dividends.
- The firm's EBIT is $13.327 million, and it faces a 40% federal-plus-state tax rate.
- The market risk premium is 5%, and the risk-free rate is 4%.
- BEA's stock price is $40 per share with 2 million shares outstanding.

We can start by calculating BEA's market value of equity and debt:

Market value of equity = Stock price * Shares outstanding = $40 * 2 million = $80 million
Market value of debt = $20 million

Based on the new capital structure, BEA's market value of equity and debt will be:

Market value of equity = Total value of the firm * Proportion of equity
= ($80 million + $22.5 million) * 0.55 = $56.375 million

Market value of debt = Total value of the firm * Proportion of debt
= ($80 million + $22.5 million) * 0.45 = $46.125 million

The amount of debt in BEA's new capital structure is $46.125 million - $20 million = $26.125 million. The amount of equity in BEA's new capital structure is $56.375 million - $26.125 million = $30.25 million.

Next, we can calculate BEA's levered beta:

βL = 0.9

D/E = $26.125 million / $30.25 million = 0.862

T = 40%

βL = βU[1 + (1 - T) * D/E]
0.9 = βU[1 + (1 - 0.4) * 0.862]
0.9 = βU[1.3012]
βU = 0.9 / 1.3012 = 0.6919

Therefore, BEA's unlevered beta is 0.69 (rounded to two decimal places).Answer: Unlevered beta = 0.69 (rounded to two decimal places). More than 100 words.

To know more about beta visit:

https://brainly.com/question/32609991

#SPJ11

what key question might an entrepreneur ask during the exploit phase of an entrepreneurial venture?

Answers

Key Question: "How can we scale our business and maximize profitability while maintaining a competitive advantage in the market?"

During the exploit phase of an entrepreneurial venture, the focus shifts towards growth and maximizing the potential of the business. The entrepreneur needs to consider various factors to ensure the long-term success of the venture. By asking the key question above, the entrepreneur seeks to identify strategies and opportunities that will allow the business to expand its operations, increase market share, and generate higher profits. They need to evaluate factors such as market demand, production scalability, cost optimization, competitive positioning, and effective marketing and sales strategies. This question helps the entrepreneur navigate the challenges of growth and make informed decisions that drive the sustainable success of the entrepreneurial venture.

Learn more about market here:

https://brainly.com/question/14418904

#SPJ11

Which statement best describes the theory of efficiency wages?
Options:
The most profitable firms keep wages low.
Above-equilibrium wages are paid by the firm in order to increase worker productivity.
Some occupations are predominantly male and others are predominantly female.
Differences in human capital explain why firms pay high wages.

Answers

The theory of efficiency wages suggests that paying higher wages can be a strategic decision for firms looking to improve productivity and profits in the long run, even if it may be more expensive in the short term.

The statement that best describes the theory of efficiency wages is that above-equilibrium wages are paid by the firm in order to increase worker productivity. The efficiency wage theory posits that paying wages above the equilibrium level in the labor market can lead to increased productivity and decreased turnover rates among workers.

This is because higher wages can motivate employees to work harder and feel more invested in their jobs, leading to greater output and profits for the firm. Additionally, paying above-equilibrium wages can attract more skilled workers to the firm, which can further enhance productivity and profitability

To know more about productivity  visit:-

https://brainly.com/question/30333196

#SPJ11

What assumption in the food industry did Howard Moskowitz change?

Answers

Answer:

"By evolving from a one-size-fits-all approach to one that embraces customer diversity, Moskowitz was able to improve customer experiences with food products from coffee to cola to spaghetti sauce and beyond."

this is what I found online

Moskowitz was able to improve consumer experiences with culinary goods ranging from coffee to soda to spaghetti sauce and beyond by shifting from a one-size-fits-all approach to one that welcomes customer variety.

What is food industry?

The food industry is a complex, global network of various businesses that provides the majority of the world's food.

The word "food industries" refers to a wide range of manufacturing, distribution, processing, and conversion operations, preperation, transport and many other facilities.

Thus, Moskowitz was able to improve consumer experiences.

For further details about food industry, click here:

https://brainly.com/question/13746091

#SPJ2


How does Sales Order Processing is connected
with Purchase Requisition Processing?

Answers

Purchase Requisition Processing involves the steps taken to procure goods or services.

Purchase requisition processing is the method of obtaining goods or services. Purchase requisition processing involves a number of stages, including identifying a requirement, preparing a requisition, approving a requisition, obtaining quotes or proposals, selecting a vendor, issuing a purchase order, and receiving the goods or services. It's critical to manage each stage in the process in order to guarantee that the correct items are procured from reputable suppliers and at the best pricing. In this manner, the company can ensure that it obtains the items it needs to operate while keeping costs down.

A request from an employee or department to purchase goods or services is called a purchase requisition. It is used to create a purchase order that is sent to a supplier following internal review and approval.

Know more about Purchase Requisition, here:

https://brainly.com/question/31927664

#SPJ11

Using the quantity equation of money describe what would happen to prices and to the real output after an expansionary monetary policy: What would happen in the short run if the money supply increases by 8%? (mark all that are correct)
- The growth rate of prices will be close to 0%
- Output increases by about 8%
- The growth rate of output will be close to 0%
- Price level increases by about 8%

Answers

The quantity equation of money describes what happens to prices and real output after an expansionary monetary policy. If the money supply increases by 8%, the price level increases by about 8 and  the growth rate of output will be close to 0%%. Therefore option (A) is the correct answer.

In economics, the quantity equation of money is used to explain the relationship between the money supply, velocity, and prices in an economy. The equation is MV = PQ, where M represents the money supply, V represents the velocity of money (i.e. the rate at which money changes hands), P represents the price level, and Q represents real output or real Gross Domestic Product (GDP).

If the money supply increases, then the quantity of money in circulation will increase. This means that there will be more money available for people to spend, which could lead to an increase in prices or real output (i.e. the quantity of goods and services produced by an economy).In the short run, if the money supply increases by 8%, then the price level will increase by about 8%.

This is because the growth rate of prices is directly proportional to the growth rate of the money supply. Output, on the other hand, may not increase by 8% because it depends on a variety of factors such as technology, capital, and labor. Therefore, the growth rate of output will be close to 0%. The growth rate of prices will be close to 0% if there is no change in the money supply. Option (A) is the correct answer.

Learn more about quantity equation of money https://brainly.com/question/32679144

#SPJ11

Other Questions
1 How were stories told in the earliest days of human civilization? they were written in booksthey were crafted in stained-glass art in religious buildingsthey were traded as currencythey were told by elders around fires at night2 Which skill-based video games mostly involve critical thinking or problem solving? puzzle gamesrole-playing gamesshooter gamesplatformer games3What is an example of an operation for a game?jumping over Bowser in a Mario gamemotion-capture devices (e.g., Kinect, WiiMote)matching three-of-a-kind to clear obstaclesachieving a high score in Pac-Man4What describes a special kind of game that is designed to mimic an experience or activity so that you can learn about it first-hand?simulationadventurerole-playingpuzzleIf u get it right u get brainliest Your boss is considering a 5-year investment project. If the project is accepted, it would require an immediate spending of $662 to buy all necessary production equipment. This equipment would be sold at the end of the project and bring your company estimated $182 in sale proceeds after taxes (or after-tax salvage value) . The team recommends immediately setting aside $60 in cash to cover any unforeseen expenses. Your boss's consulting team also estimated that the annual after-tax profits (or operating cash flows) would equal $173 The required annual rate of return is 10.7%. What is the Net Present Value of this proposed investment project? If your answer is negative, don't forget the minus sign! Increase decimal places for any intermediate calculations, from the default 2 to 6 or higher. TRUE/FALSE: When only water molecules diffuse through a cell's membrane, the process is referred to as hydrolysis. Please help me please with both questions if janae notices that each apple she eats through the day yields a smaller amount of additional satisfaction, she is noticing the law of: The complex numbers $z$ and $w$ satisfy $|z| = |w| = 1$ and $zw \ne -1.$(Prove that $\overline{z} = \frac{1}{z}$ and $\overline{w} = \frac{1}{w}.$ Frogs and toads belong to the order Anura. The smallest organism in this order is about 7 millimeters long, while the largest member is about 30centimeters long. Which of these lists best describes this order? Select all that identify a covalent bond.metal - metalmetal - nonmetalnonmetal - nonmetalmetal - polyatomicpolyatomic - polyatomicpolyatomic - nonmetal Grate Ltd Co produces shoes and their short-run estimated production function is given as: Q = 90L +24L - 3L a. After what number of labour does average product start to decline?b. At what number of labours will marginal product be maximized? So, how much is the maximum marginal product? c. If price of output is $20/unit and wages paid to labour is $400; what should be the most optimum number of labour employed. HELP FAST PLS Write one paragraph discussing the impact of imperialism on China. Use the information you learned in the lesson about the impact of colonization on China in your response. [1] An uneasy peace prevailed between the Russians and the other Allies, but each side was suspicious of the other. [2] The United States wanted desperately to know what the Soviets were thinking. [3] The big fear for the U.S. was that the Soviets would attack West Germany without warning, or in diplomatic language, with "a cold start." [4] The U.S. knew that the Soviets possessed nuclear weapons. [5] Would they use them on the West? [6] This question and others would need to be answered by U.S. intelligence agents. The Dark Game, Paul Janeczko Use the drop-down menus to answer questions about the sentence structures in this paragraph. What is the structure of sentence 1? What is the structure of sentence 3? What is the structure of sentence 6? Please help no links please Which of the following are cited in your text as reasons for choosing abortion? a. the mother feels that she is too young to care for a child b. medical conditions c. all are correct d. the family is not financially able to support the child Every traveller is a tourist but every tourist is not a traveller justify fill in the blanks [tex] {(x + 3)}^{2} - x - 3 = (x + 3) \times ............[/tex]please answer my question I will mark you as brainliest and I will follow you Work out 5/6 x 3/4Give the answer as a fraction in its simplest form. 10111011/111 binary division Bethany, who weighs 460 N, lies in a hammock suspended by ropes tied to two trees. The left rope makes an angle of 45 with the ground; the right one makes an angle of 30.Find the tension in the left rope.Find the tension in the right rope. Please help me with this questions please please ASAP ASAP please ASAP help please please ASAP please I'm begging you please please ASAP help me please:(basically 50 points